Another triple integral problem

Click For Summary
The discussion revolves around evaluating the triple integral of the function 1/sqrt(x^2+y^2+(z-A)^2) over a spherical domain defined by x^2+y^2+z^2 ≤ R^2. The integrand suggests a change to spherical coordinates, leading to a new coordinate system centered at (0,0,A). The boundaries for the radial coordinate ρ are determined to be A-R ≤ ρ ≤ A+R, while the angular coordinate θ ranges from 0 to 2π. The user initially struggles with solving the integral but finds that switching to cylindrical coordinates simplifies the process, allowing for easier integration. Ultimately, the integral is successfully solved after this adjustment.
Gianmarco
Messages
42
Reaction score
3

Homework Statement


Calculate
<br /> \int_D \frac{dxdydz}{\sqrt{x^2+y^2+(z-A)^2}}, \: A&gt;R<br />
on ## D = {(x,y,z)\: s.t. x^2+y^2+z^2 \leq R^2}##.

Homework Equations


In spherical coordinates:
<br /> x=\rho cos\theta sin\phi\\y=\rho sin\theta sin\phi\\ z=cos\phi\\dxdydz=\rho^2sin\phi d\theta d\rho d\phi<br />

The Attempt at a Solution


Since the integrand is ##\frac{1}{\rho}## of a sphere centered in ##(0,0,A)##, I think the most logical change of coordinates is:
<br /> x=\rho cos\theta sin\phi\\y=\rho sin\theta sin\phi\\ z=\rho cos\phi + A<br />
I am centered at a point ##(0,0,A)##, so nothing changed with respect to the xy-plane, therefore ##0 \leq \theta \leq 2\pi##; that much is obvious. And, since A>R, the boundaries for ##\rho## are ##A-R \leq \rho \leq A+R##. I found those geometrically.
Changing the variables to##\rho, \theta, \phi## in D we get:
##\rho^2 + 2\rho Acos\phi + A^2 \leq R^2##
And at this point I have no idea. Solving for ##\phi## would give me an upper limit, but it would be a an arccos of the form ##arccos(\frac{a}{\rho} - b\rho)## which I'd have to integrate in ##\rho## afterwards. Any ideas?
 
Physics news on Phys.org
I have got the impression that the cylindrical coordinate is more helpful instead. In this case, the integral will be
$$
\int_0^{2\pi} \int_{-R}^R \int_0^{r(z)} \frac{rdrd\phi dz}{\sqrt{r^2+(z-A)^2}}
$$
Performing the integral over ##r## first should be easy.
 
  • Like
Likes Gianmarco
I was able to solve it, thank you for the hint blue leaf! :)
 
Question: A clock's minute hand has length 4 and its hour hand has length 3. What is the distance between the tips at the moment when it is increasing most rapidly?(Putnam Exam Question) Answer: Making assumption that both the hands moves at constant angular velocities, the answer is ## \sqrt{7} .## But don't you think this assumption is somewhat doubtful and wrong?

Similar threads

  • · Replies 2 ·
Replies
2
Views
2K
Replies
3
Views
2K
  • · Replies 34 ·
2
Replies
34
Views
3K
  • · Replies 4 ·
Replies
4
Views
2K
  • · Replies 14 ·
Replies
14
Views
2K
  • · Replies 7 ·
Replies
7
Views
2K
  • · Replies 1 ·
Replies
1
Views
2K
  • · Replies 6 ·
Replies
6
Views
2K
Replies
2
Views
1K
  • · Replies 3 ·
Replies
3
Views
3K